1965 AHSME Problems/Problem 19

Revision as of 16:30, 4 September 2021 by Thisusernameistaken (talk | contribs)

Problem 19

If $x^4 + 4x^3 + 6px^2 + 4qx + r$ is exactly divisible by $x^3 + 3x^2 + 9x + 3$, the value of $(p + q)r$ is:

$\textbf{(A)}\ - 18 \qquad  \textbf{(B) }\ 12 \qquad  \textbf{(C) }\ 15 \qquad  \textbf{(D) }\ 27 \qquad  \textbf{(E) }\ 45 \qquad$

Solution 1

See Also

1965 AHSC (ProblemsAnswer KeyResources)
Preceded by
Problem 39
Followed by
Last Question
1 2 3 4 5 6 7 8 9 10 11 12 13 14 15 16 17 18 19 20 21 22 23 24 25 26 27 28 29 30 31 32 33 34 35 36 37 38 39 40
All AHSME Problems and Solutions

The problems on this page are copyrighted by the Mathematical Association of America's American Mathematics Competitions. AMC logo.png